Đến nội dung

hungvu11

hungvu11

Đăng ký: 19-09-2010
Offline Đăng nhập: 29-03-2014 - 09:55
-----

#454825 Đề thi chọn học sinh giỏi toán lớp 11-12 chuyên KHTN 2013-2014 (Vòng 1)

Gửi bởi hungvu11 trong 03-10-2013 - 13:06

Bạn có thể giới thiệu cách sử dụng định lí pascal được không? Thanks

Thực ra cách của mình tương tự bạn perfectstrong. Còn cách sử dụng pascal là của bạn mình mình sẽ hỏi lại sau.

Bài 3 ngày 2 ngoài cách sử dụng định lý Thales ta có thể gọi $EK \cap AB =M, EL \cap AC = N$. 

Bằng tính toán và sử dụng giả thiết ta chứng minh được AM = AN.

Từ đó chứng minh được EK và PQ cùng vuông góc với AI (I là tâm nội tiếp)




#453970 Đề thi chọn học sinh giỏi toán lớp 11-12 chuyên KHTN 2013-2014 (Vòng 1)

Gửi bởi hungvu11 trong 29-09-2013 - 19:09



Đề có vấn đề?

ScreenHunter_05%20Sep.%2029%2019.01.gif

thầy ơi có điều kiện của  $\angle MNQ$  nữa

Thực sự mình rất ấn tượng với 2 bài hình này.

Bài 1 có thể sử dụng định lý Pascal hoặc chỉ cần dùng tứ giác nội tiếp

Bài 2 cũng rất hay vì bài có điều kiện khá là ảo




#438325 IMO 2013

Gửi bởi hungvu11 trong 26-07-2013 - 11:46

Bài 5:

1. Ta sẽ chứng minh $f(x)$ tăng

Từ ii/ theo quy nạp ta có $f(na) \geq nf(a) = na$

Từ i/ ta có $f(n)f(a) \geq f(na) \geq na$ do đó $f(n) \geq n$ với mọi $n \in N^{*}$

Với $x \in Q^{+}$ tồn tại $n \in N^{*}$ sao cho $nx=m \in N^{*}$ 

Thay vào i/ ta có $f(n)f(x) \geq f(m)$ do đó $f(x) \geq 0$

Nên từ ii/ $f$ là hàm tăng

2. Chưng minh $f(x) = x$ với mọi $x \in N^{*}$

Từ i/ theo quy nạp ta có $ a^{n} \geq f(a^{n})$ với $n \in N^{*}$

Giả sử tồn tại $m \in N^{*}$ sao cho $f(m) = m +\varepsilon (\varepsilon >0)$

Khi đó tồn tại $N$ sao cho với $k>N$ thì $ k\varepsilon > m$

Do $ a>1$ nên $a^{n} \rightarrow \infty$ khi $n \rightarrow \infty$

Nên tồn tại $M$ sao cho  với $n>M$ thì $a^{n} \geq mN$

Khi đó cho $n>M$ tồn tại $p \in N^{*}$ sao cho $ 0 \leqslant a^{n} - pm < m$ với $p>N (1)$

Do f là hàm tăng nên $ a^{n} \geq f(a^{n}) \geq f(pm) \geq pm + p \varepsilon \geq pm + m $ do $p>N$ mâu thuẫn với $(1)$

Nên $f(m)=m$ với mọi $m \in N{*}$

Từ i/ và ii/ suy ra $mf(y) \geq f(my) \geq m.f(y)$

nên $mf(y)= f(my)$ với mọi $m \in N^{*}, y \in Q^{+}$

Từ đó suy ra $f(x)=x$ với mọi $x \in Q^{+}$.

 

 

 

 

 

 

 

 




#272311 Tản mạn BĐT

Gửi bởi hungvu11 trong 13-08-2011 - 20:41

Bài 37: (1 bài đơn giản)
Cho $a,b,c > 0$. Chứng minh: $\sum\limits_{cyc} {\dfrac{{a^9 }}{{bc}}} + \dfrac{2}{{abc}} \ge \sum\limits_{cyc} {a^5 + 2} $

Áp dụng bất đẳng thức AM-GM ta có:
$ \sum\limits_{cyc} {\dfrac{ a^{9} }{bc} + abc \geq 2 \sum\limits_{cyc} {\sqrt{ \dfrac{a^{9}}{bc} . abc } = \sum\limits_{cyc} {{2a^{5}} $
$ \dfrac{2}{abc} + 2abc \geq 4 $
$ \Rightarrow \sum\limits_{cyc} {\dfrac{{a^9 }}{{bc}}} + \dfrac{2}{{abc}} +5abc \geq 2(a^{5} + b^{5} +c^{5})+4 $ (1)
Lại có:
$ a^{5} + b^{5} + c^{5} + 1 + 1 \geq 5abc $(2)
Từ (1) và (2) ta có đpcm